3
$\begingroup$

While reading about the almost-splitting theorem of Cheeger and Colding, I thought about the following question, which asks about wether the level sets of a function which is close to being a linear form in an average sense behave like the level sets of a linear form.

It seems to me that the end of the proof of the almost splitting could be made easier to follow by explicitly using the lemma I am statin below, however I had no luck trying to come up with a proof so far.

The lemma that I would like to use (in a simplified version) is the following :

Lemma :

For any $\varepsilon>0$, there is an $\eta>0$ such that if $u:B(0,2)\subset\mathbb{R}^n\to\mathbb{R}$ is a smooth function with :

  • $\int_{B(0,2)}\left||\nabla u|^2-1\right|\leq\eta$,
  • $\int_{B(0,2)}\left|\mathrm{Hess}\, u\right|^2\leq\eta$,

then the set of $(x,y)\in B(0,1)\times B(0,1)$ for which

$\Big | |u(y)-u(x)|-d\left(y,\{u=u(x)\}\right)\Big|>\varepsilon$

has measure less than $\varepsilon$. Here $d\left(y,\{u=u(x)\}\right)= \inf\Big\{d(z,y)\,\Big |\, z\in B(0,R),\,u(z)=u(x)\Big\}$.

I know how to prove it if the $L^2$ bounds are replaced with $L^\infty$ bounds. I also know how to prove that the set of $(x,y)\in B(0,1)\times B(0,1)$ such that

$\left |u(y)-u(x)-\langle\nabla u(x),y-x\rangle\right|\geq \varepsilon\|y-x\|$

has measure less than $\varepsilon$ provided $\eta$ is small enough.

If needed, feel free to assume the integral bounds on balls bigger than $B(0,2)$.

Note : The real lemma I need is stated in terms of functions defined on balls in a Riemannian manifold with $\mathrm{Ric}\geq -\eta g$. I chose to state it for function in $\mathbb{R}^n$ because :

  • the heart of the difficulty remains in this setting.
  • it may look more appealing to PDE specialists not too fond of differential geometry.
$\endgroup$
2
  • $\begingroup$ This question is interesting to me. Could you add a proof for $L_\infty$ case and a proof for additional problem $\left |u(y)-u(x)-\langle\nabla u(x),y-x\rangle\right|\geq \varepsilon\|y-x\|$ ? $\endgroup$ Mar 31, 2017 at 2:48
  • 1
    $\begingroup$ I'll have to get to my notes and that might take me a while, I'll try to not forget it. $\endgroup$ Apr 6, 2017 at 14:28

0

Your Answer

By clicking “Post Your Answer”, you agree to our terms of service and acknowledge you have read our privacy policy.